Diễn Đàn MathScopeDiễn Đàn MathScope
  Diễn Đàn MathScope
Ghi Danh Hỏi/Ðáp Community Lịch

Go Back   Diễn Đàn MathScope > Sơ Cấp > Việt Nam và IMO > 2015

News & Announcements

Ngoài một số quy định đã được nêu trong phần Quy định của Ghi Danh , mọi người tranh thủ bỏ ra 5 phút để đọc thêm một số Quy định sau để khỏi bị treo nick ở MathScope nhé !

* Nội quy MathScope.Org

* Một số quy định chung !

* Quy định về việc viết bài trong diễn đàn MathScope

* Nếu bạn muốn gia nhập đội ngũ BQT thì vui lòng tham gia tại đây

* Những câu hỏi thường gặp

* Về việc viết bài trong Box Đại học và Sau đại học


Trả lời Gởi Ðề Tài Mới
 
Ðiều Chỉnh Xếp Bài
Old 26-03-2015, 02:11 AM   #1
huynhcongbang
Administrator

 
huynhcongbang's Avatar
 
Tham gia ngày: Feb 2009
Đến từ: Ho Chi Minh City
Bài gởi: 2,413
Thanks: 2,165
Thanked 4,188 Times in 1,381 Posts
Gửi tin nhắn qua Yahoo chát tới huynhcongbang
Dưới đây là lời giải chi tiết cho bài hình. Theo mình thì bài này tuy có mô hình khá đơn giản nhưng lại kết hợp nhiều kết quả có sẵn, nếu không nắm vững sẽ giải rất vất vả. Mình phải mất gần 2 tiếng mới giải xong bài này.


a) Gọi $E$ là điểm đối xứng với $A$ qua $O$ thì dễ thấy tứ giác $BHCE$ là hình bình hành và có $K,H,I,E$ thẳng hàng.

Do đó $\angle AKI=\angle ADI=90{}^\circ $ nên tứ giác $AKDI$ nội tiếp. Suy ra $$\angle AIB=180{}^\circ -\angle AKD=\angle ACM.$$ Ngoài ra, $\angle ABI=\angle AMC$ nên hai tam giác $ABI,AMC$ đồng dạng, suy ra $\angle BAI=\angle MAC$.

Gọi $F$ là giao điểm của trung tuyến $AI$ với $(O)$ thì từ các tam giác đồng dạng, ta có $$\angle FCB=\angle BAI=\angle MAC=\angle CBM.$$ Do đó, $BM=CF$ hay $MF\parallel BC.$

Suy ra $IM=IF=IN$ nên $BC$đi qua trung điểm của $MN$ hay nói cách khác, $M,N$ đối xứng nhau qua $BC$.

Vậy $\angle BNC=\angle BMC=180{}^\circ -\angle BAC$ cố định nên điểm $N$ di chuyển trên cung chứa góc $180{}^\circ -\angle BAC$ dựng trên $BC$, tức là thuộc đường tròn cố định.

b) Gọi $(S)$ là đường tròn ngoại tiếp tam giác $APQ.$ Giả sử $HN$ cắt $(S)$ tại $R$.

Tứ giác $HNEJ$ có hai đường chéo cắt nhau tại trung điểm mỗi đường nên là hình bình hành, suy ra $HN\parallel JE$, mà $JE\bot AJ$ nên $HN\parallel AJ.$ Do đó, tứ giác $HNID$ nội tiếp.

Ta có $$\angle AQN=\angle ARN=\angle NHI=\angle NDI$$ nên $CQND$ nội tiếp.

Suy ra $$\angle NQD=\angle NCD=\angle DCM=\angle BAM=\angle CAJ=\angle QAN$$ hay $DQ$ là tiếp tuyến của đường tròn $(S).$
Tương tự thì $DP$ cũng là tiếp tuyến của $(S)$ nên $AD$ là đường đối trung của tam giác $APQ$, tức là $AJ,AD$ đối xứng nhau qua phân giác góc $\angle BAC.$

Hơn nữa, do $H,O$ lần lượt là trực tâm và tâm đường tròn ngoại tiếp tam giác $ABC$ nên $AH,AO$ cũng đối xứng nhau qua phân giác góc $\angle BAC$.

Từ đây suy ra $AJ,AO$ trùng nhau hay $AJ$ đi qua điểm $O$ cố định.
[RIGHT][I][B]Nguồn: MathScope.ORG[/B][/I][/RIGHT]
 
Hình Kèm Theo
Kiểu File : png abc.png (60.3 KB, 1465 lần tải)
__________________
Sự im lặng của bầy mèo

thay đổi nội dung bởi: huynhcongbang, 26-03-2015 lúc 09:40 AM
huynhcongbang is offline   Trả Lời Với Trích Dẫn
The Following 5 Users Say Thank You to huynhcongbang For This Useful Post:
alibaba_cqt (31-03-2015), dangvip123tb (01-04-2015), quocbaoct10 (26-03-2015), thiendieu96 (27-03-2015), vantienducdh (26-03-2015)
Old 26-03-2015, 06:11 AM   #2
quocbaoct10
+Thành Viên Danh Dự+
 
quocbaoct10's Avatar
 
Tham gia ngày: Oct 2012
Đến từ: THPT chuyên Lê Quý Đôn-Nha Trang-Khánh Hòa
Bài gởi: 539
Thanks: 292
Thanked 365 Times in 217 Posts
Nhận xét một chút về bài 3:
Từ các con số $20$ hay $20^{15}$, ta hoàn toàn có thể tổng quát nó lên thành các số $m=a.b$ với $a=2^k$ và $b$ là một số có căn nguyên thủy, cụ thể $2,4,p^k,2p^k$. Lời giải Hoàn toàn có thể xây dựng dựa vào bổ đề 1 trong lời giải của anh Trung :"Cho $p$ là một số nguyên tố lẻ khi đó $S_k(p)$ không chia hết cho $p$ khi và chỉ khi $(p - 1) | k$". Và có một câu hỏi được đặt ra: liệu với $m$ là tích của 2 số có căn nguyên thủy( 2 số này khác $2$ và $4$) thì có tồn tại $k \neq 1$ sao cho $k$ mang tính chất $t-m$ hay không ? Và nếu chứng minh được điều này thì việc mở rộng ra với tất cả các số tự nhiên là hoàn toàn có thể, dựa vào bổ đề 3 của anh Trung: "Cho $m, n$ là 2 số nguyên dương nguyên tố cùng nhau. Khi đó nếu $k$ có tính chất "$t-mn$" khi và chỉ khi nó có tính chất "$t-m$" và "$t-n$" ".
[RIGHT][I][B]Nguồn: MathScope.ORG[/B][/I][/RIGHT]
 
__________________
i'll try my best.

thay đổi nội dung bởi: quocbaoct10, 26-03-2015 lúc 06:25 AM
quocbaoct10 is offline   Trả Lời Với Trích Dẫn
The Following 5 Users Say Thank You to quocbaoct10 For This Useful Post:
dangvip123tb (01-04-2015), Raul Chavez (26-03-2015), thaygiaocht (26-03-2015), thiendieu96 (27-03-2015), vantienducdh (26-03-2015)
Old 26-03-2015, 10:56 AM   #3
huynhcongbang
Administrator

 
huynhcongbang's Avatar
 
Tham gia ngày: Feb 2009
Đến từ: Ho Chi Minh City
Bài gởi: 2,413
Thanks: 2,165
Thanked 4,188 Times in 1,381 Posts
Gửi tin nhắn qua Yahoo chát tới huynhcongbang
Gửi mọi người đề ngày 1 mình đã gõ Latex xong.

Chờ đề ngày 2 thôi...
[RIGHT][I][B]Nguồn: MathScope.ORG[/B][/I][/RIGHT]
 
File Kèm Theo
Kiểu File : pdf VNTST 2015 - Day 1.pdf (106.6 KB, 367 lần tải)
__________________
Sự im lặng của bầy mèo
huynhcongbang is offline   Trả Lời Với Trích Dẫn
The Following 6 Users Say Thank You to huynhcongbang For This Useful Post:
dangvip123tb (26-03-2015), n.t.tuan (26-03-2015), n.v.thanh (27-03-2015), quocbaoct10 (26-03-2015), thaygiaocht (26-03-2015), thiendieu96 (27-03-2015)
Old 26-03-2015, 01:34 PM   #4
n.t.tuan
+Thành Viên+
 
n.t.tuan's Avatar
 
Tham gia ngày: Nov 2007
Bài gởi: 1,250
Thanks: 119
Thanked 616 Times in 249 Posts
Đề ngày 2 đâu các bạn ơi? Thấy bảo bài Đại số khó lắm.
[RIGHT][I][B]Nguồn: MathScope.ORG[/B][/I][/RIGHT]
 
__________________
T.
n.t.tuan is offline   Trả Lời Với Trích Dẫn
The Following User Says Thank You to n.t.tuan For This Useful Post:
dangvip123tb (26-03-2015)
Old 26-03-2015, 02:24 PM   #5
huynhcongbang
Administrator

 
huynhcongbang's Avatar
 
Tham gia ngày: Feb 2009
Đến từ: Ho Chi Minh City
Bài gởi: 2,413
Thanks: 2,165
Thanked 4,188 Times in 1,381 Posts
Gửi tin nhắn qua Yahoo chát tới huynhcongbang
Dưới đây là đề thi ngày 2, mình xin được từ bạn Lê Nhật Hoàng (LNH).

Bài 4.

Trong một kỳ thi vấn đáp, có $100$ thí sinh và $25$ vị giám khảo, mỗi thí sinh thích ít nhất $10$ giám khảo.

a) Chứng minh rằng có thể chọn ra $7$ giám khảo mà mỗi thí sinh đều thích ít nhất $1$ trong $7$ người đó.
b) Chứng minh rằng có thể sắp xếp lịch thi sao cho mỗi thí sinh được đúng $1$ giám khảo mình thích hỏi và mỗi giám khảo hỏi không quá $10$ thí sinh.

Bài 5.

Cho tam giác$ABC$ nhọn và có điểm $P$ nằm trong tam giác sao cho $\angle APB=\angle APC = \alpha$ và $\alpha>180{}^\circ - \angle BAC $. Đường tròn ngoại tiếp tam giác $APB$ cắt $AC$ ở $E,$ đường tròn ngoại tiếp tam giác $APC$ cắt $AB$ ở $F$ . Gọi $Q$ là điểm nằm trong tam giác $AEF$ sao cho $\angle AQE=\angle AQF$. Gọi $D$ là điểm đối xứng với $Q$ qua $EF$ , phân giác góc $EDF$ cắt $AP$ tại $T.$

a) Chứng minh rằng $\angle DET=\angle ABC,\angle DFT=\angle ACB$ .
b) Đường thẳng $PA$ cắt các đường thẳng $DE,DF$ lần lượt tại $M,N$ . Gọi $I,J$ lần lượt là tâm đường tròn nội tiếp các tam giác $PEM,PFN$ và $K$ là tâm đường tròn ngoại tiếp tam giác $DIJ$ . Đường thẳng $DT$ cắt $(K)$ tại $H$. Chứng minh rằng $HK$ đi qua tâm đường tròn nội tiếp của tam giác $DMN.$

Bài 6.

Tìm số nguyên dương $n$ nhỏ nhất sao cho tồn tại $n$ số thực thỏa mãn đồng thời các điều kiện:

i) Tổng của $n$ số đó dương.
ii) Tổng lập phương của $n$ số đó âm.
iii) Tổng lũy thừa bậc $5$ của $n$ số đó dương.

----------------------

Dưới đây là file Latex cho ngày 2.
[RIGHT][I][B]Nguồn: MathScope.ORG[/B][/I][/RIGHT]
 
File Kèm Theo
Kiểu File : pdf VNTST 2015 - Day 2.pdf (102.1 KB, 299 lần tải)
__________________
Sự im lặng của bầy mèo

thay đổi nội dung bởi: huynhcongbang, 26-03-2015 lúc 02:46 PM
huynhcongbang is offline   Trả Lời Với Trích Dẫn
The Following 10 Users Say Thank You to huynhcongbang For This Useful Post:
dangvip123tb (26-03-2015), Mộc Gia San (26-03-2015), n.t.tuan (26-03-2015), n.v.thanh (27-03-2015), quocbaoct10 (26-03-2015), thanhgand (28-03-2015), thaygiaocht (26-03-2015), thiendieu96 (27-03-2015), vantienducdh (26-03-2015), whatever2507 (26-03-2015)
Old 26-03-2015, 03:43 PM   #6
thaygiaocht
+Thành Viên+
 
thaygiaocht's Avatar
 
Tham gia ngày: Aug 2012
Đến từ: Chuyên Hà Tĩnh
Bài gởi: 165
Thanks: 793
Thanked 216 Times in 93 Posts
Cảm nhận đầu tiên là đề năm nay có hình thức phát biểu bài toán đẹp hơn và có mức độ khó hơn mọi năm.
[RIGHT][I][B]Nguồn: MathScope.ORG[/B][/I][/RIGHT]
 
__________________
https://www.facebook.com/thaygiaocht
thaygiaocht is offline   Trả Lời Với Trích Dẫn
The Following 3 Users Say Thank You to thaygiaocht For This Useful Post:
dangvip123tb (26-03-2015), thiendieu96 (27-03-2015), vantienducdh (26-03-2015)
Old 26-03-2015, 04:18 PM   #7
Nvthe_cht.
+Thành Viên+
 
Tham gia ngày: Nov 2013
Bài gởi: 69
Thanks: 15
Thanked 36 Times in 24 Posts

Đây là lời giải bài hình ngày 1 của mình.
Gọi $T$ là giao điểm của $BC$ và $EF$. $N'$ là giao điểm của $TH$ với $AI$, $M'$ là điểm đối xứng của $N'$ qua $BC$.
Theo các kết quả quen thuộc thì ta có $T,K,A$ thẳng hàng. $TH$ vuông góc $AI$.
Ta có: $\widehat{M'DI}=\widehat{N'DI}=\widehat{TAN}=\wide hat{TDK}$
Nên $K, D, M$ thẳng hàng.
Mặt khác không khó để có $BHN'C$ nội tiếp nên $M'$ thuộc đường tròn $(O)$
Từ các điều trên ta có $M'$ trùng $M$ hay $N'$ trùng $N$, nói cách khác $N$ nằm trên đường tròn $(BHC)$ cố định.
Chú ý: $(BHC)$ cố định vì $\widehat{BHC}$ không đổi và $BC$ cố định.

b. Gọi $S$ là giao điểm của $PQ$ với $AT$. Khi đó $A$ nằm trên đường đối cực của $S$ với đường tròn $(APQ)$, mặt khác chú ý rằng $A(SD,PQ)=-1$ nên $AD$ là đường đối cực của $S$ đối với $(APQ)$, hay $PP,QQ,AD$ đồng quy, từ đây thì ta có $AJ,AD$ đẳng giác trong góc $\widehat{PAQ}$, do vậy $AJ$ đi qua tâm O cố định.
[RIGHT][I][B]Nguồn: MathScope.ORG[/B][/I][/RIGHT]
 
Nvthe_cht. is offline   Trả Lời Với Trích Dẫn
The Following 2 Users Say Thank You to Nvthe_cht. For This Useful Post:
dangvip123tb (26-03-2015), sieusieu90 (26-03-2015)
Old 26-03-2015, 05:21 PM   #8
huynhcongbang
Administrator

 
huynhcongbang's Avatar
 
Tham gia ngày: Feb 2009
Đến từ: Ho Chi Minh City
Bài gởi: 2,413
Thanks: 2,165
Thanked 4,188 Times in 1,381 Posts
Gửi tin nhắn qua Yahoo chát tới huynhcongbang
Bài cuối thì mình mới có một số ý thế này:

Dễ thấy $n=1$ là không thỏa.

- Với $n=2$, giả sử ta có 2 số $a,b$ thì phải có hệ điều kiện $$\left\{\begin{matrix}
a+b>0\\ a^3+b^3<0
\\ a^5+b^5>0
\end{matrix}\right.$$ Dễ thấy điều này cũng không xảy ra được vì $a^3+b^3=(a+b)(a^2-ab+b^2)$ là luôn cùng dấu với $a+b$.

- Với $n=3$, xét 3 số $(a,b,c)$, ta có hệ điều kiện $$\left\{\begin{matrix}
a+b+c>0\\ a^3+b^3+c^3<0
\\ a^5+b^5+c^5>0
\end{matrix}\right.$$ Ta sẽ chứng minh điều này là không thể xảy ra.

Rõ ràng trong 3 số phải có ít nhất 1 số âm và ít nhất 1 số dương. Ta xét 2 trường hợp:

+ Nếu có 2 số âm, 1 số dương, giả sử là $a,b<0, c > 0$ thì đặt $a'= -a>0, b' = -b$, ta có hệ mới $$\left\{\begin{matrix}
a'+b'<c\\ a'^3+b'^3>c^3
\\ a'^5+b'^5<c^5
\end{matrix}\right.$$ Nhân BĐT thứ nhất và thứ hai, ta có $(a'+b')(a'^5+b'^5)<c'^6 = (c'^3)^2 < (a'^3+b'^3)^2$, sai theo BĐT Cauchy-Schwarz.

+ Nếu có 2 số dương, 1 số âm, giả sử là $a,b>0, c < 0$ thì đặt $c'= -c>0$, ta có hệ mới $$\left\{\begin{matrix}
a+b>c'\\ a^3+b^3<c'^3
\\ a^5+b^5>c'^5
\end{matrix}\right.$$ Đến đây ta cũng chứng minh được BĐT này sai.

- Với $n=5$, ta chỉ ra được một bộ thỏa mãn là $$(a,b,c,d,e) = (-7,-7,2,5,8).$$

Do đó, chỉ còn trường hợp $n=4$ (mình đoán là không thỏa, nhưng chưa chứng minh được).
[RIGHT][I][B]Nguồn: MathScope.ORG[/B][/I][/RIGHT]
 
__________________
Sự im lặng của bầy mèo
huynhcongbang is offline   Trả Lời Với Trích Dẫn
The Following 3 Users Say Thank You to huynhcongbang For This Useful Post:
dangvip123tb (26-03-2015), thiendieu96 (27-03-2015), vantienducdh (26-03-2015)
Old 26-03-2015, 06:43 PM   #9
Traum
Moderator
 
Traum's Avatar
 
Tham gia ngày: Nov 2007
Đến từ: cyber world
Bài gởi: 413
Thanks: 14
Thanked 466 Times in 171 Posts
Bài 6: Chứng minh n = 4 không thỏa.

Ta có $a + b + c + d > 0$, $a^3 + b^3 + c^3 + d^3 < 0$ và $a^5 + b^5 + c^5 + d^5 > 0$. Nên trong 4 số $a$, $b$, $c$, $d$ có ít nhất 1 số âm và 1 số dương.

Xét 3 trường hợp:
TH1: $b > 0$, $c > 0$.
Khi đó ta thay $d$ bởi $-d$ thì điều kiện có thể viết lại thành: $a + b + c > d$, $a^3 + b^3 + c^3 < d^3$, $a^5 + b^5 + c^5 > d^5$.
Gọi $t > 1$ sao cho $t^3(a^3 + b^3 + c^3) = d^3$. Khi đó nếu thay $a$, $b$, $c$ bởi $a_1 = at$, $b_1 = bt$, $c_1 = ct$ thì ta có:
$a_1 + b_ 1 + c_1 = t(a+b+c) > d$, $a_1^3 + b_1^3 + c_1^3 = d^3$ và $a_1^5 + b_1^5 + c_1^5 = t^5(a^5 + b^5 + c^5) > d^5$.
Tuy nhiên khi đó thì vì $a_1, b_1, c_1 < d$ nên $1 = \left(\frac{a_1}{d}\right)^3 + \left(\frac{b_1}{d}\right)^3 + \left(\frac{c_1}{d}\right)^3 > \left(\frac{a_1}{d}\right)^5 + \left(\frac{b_1}{d}\right)^5 + \left(\frac{c_1}{d}\right)^5 > 1$. Vô lý.

TH2: $b < 0$, $c < 0$ tương tự TH1.

TH3: $b > 0$, $c < 0$.
Thay $c$ bởi $-c$, ta viết lại thành $a+b > c+d$, $a^3 + b^3 < c^3 + d^5$, $a^5 + b^5 > c^5 + d^5$ với $a, b, c, d > 0$. Khi đó tương tự như TH1, TH2 ta xét thay $a$, $b$ bởi $at$, $bt$ với $t>1$ sao cho $a^3 + b^3 = c^3 + d^3$.
Giả sử $a \ge b$, $c \ge d$.
TH3a. $a$ là số lớn nhất hay $a > c$. Khi đó $a^3 > c^3 \ge d^3 > b^3$. Ta có hàm số $f(x) = x^{\frac{1}{3}}$ là hàm lõm trên $R^+$ nên $f(a^3) + f(b^3) \le f(c^3) + f(d^3)$ hay $a + b \le c + d$. Vô lý.
TH3b. $c$ là số lớn nhất hay $c > a$. Khi đó $c^3 > a^3 > b^3 > d^3$. Ta có hàm số $f(x) = x^{\frac{5}{3}}$ là hàm lồi trên $R^{+}$ nên $f(a^3) + f(b^3) \le f(c^3) + f(d^3)$ hay $a^5 + b^5 \le c^5 + d^5$. Vô lý.

Vậy ta có $n=4$ không thỏa. Hiển nhiên $n=2,3$ cũng không thỏa. Với $n=5$ thì lấy như post trên $-7, -7, 2, 5, 8$ thỏa mãn.
[RIGHT][I][B]Nguồn: MathScope.ORG[/B][/I][/RIGHT]
 
__________________
Traum is giấc mơ.

thay đổi nội dung bởi: Traum, 26-03-2015 lúc 07:07 PM
Traum is offline   Trả Lời Với Trích Dẫn
The Following 5 Users Say Thank You to Traum For This Useful Post:
dangvip123tb (01-04-2015), huynhcongbang (27-03-2015), quocbaoct10 (26-03-2015), thaygiaocht (26-03-2015), vantienducdh (26-03-2015)
Old 26-03-2015, 07:03 PM   #10
Traum
Moderator
 
Traum's Avatar
 
Tham gia ngày: Nov 2007
Đến từ: cyber world
Bài gởi: 413
Thanks: 14
Thanked 466 Times in 171 Posts
Trích:
Nguyên văn bởi chemthan View Post
Hình như TH3 sai rồi @, thấy không sử dụng đến $a^5 + b^5 > c^5 + d^5$. Chỗ giả sử $a$ lớn nhất là sai, $a$ chỉ bình đẳng với $b$ thôi!
Làm vội nên quên mất vai trò của của chúng. Nếu a không lớn nhất thì phải sử dụng điều kiện cho lũy thừa 5. Như trên.
[RIGHT][I][B]Nguồn: MathScope.ORG[/B][/I][/RIGHT]
 
__________________
Traum is giấc mơ.
Traum is offline   Trả Lời Với Trích Dẫn
The Following User Says Thank You to Traum For This Useful Post:
thaygiaocht (26-03-2015)
Old 26-03-2015, 06:49 PM   #11
quocbaoct10
+Thành Viên Danh Dự+
 
quocbaoct10's Avatar
 
Tham gia ngày: Oct 2012
Đến từ: THPT chuyên Lê Quý Đôn-Nha Trang-Khánh Hòa
Bài gởi: 539
Thanks: 292
Thanked 365 Times in 217 Posts
Bài 4
a) đặt $A_i$ là tập các giám khảo mà học sinh thứ $i$ thích, ta có $|A_i| \ge 10$.
Khi đấy theo nguyên lí Dirichlet, ta tìm được 1 giám khảo được thích bởi ít nhất $\frac{\sum^{100}_{i=1} |A_i|}{25} =40 $. Bỏ đi 40 học sinh này và vị giám khảo được thích bởi 40 học sinh, ta được số học sinh còn lại là 60 và số giám khảo còn lại là 24. Tiếp tục áp dụng nguyên lí Dirichlet 1 cách liên tục, ta sẽ tìm được 7 vị giáo khảo thỏa ycđb.
b) Ta sẽ tìm cách ghép các học sinh thành nhiều nhất là 25 nhóm và ít nhất là 10 nhóm sao cho mỗi nhóm này thích một giám khảo khác nhau. Ta chỉ cần chứng minh được trường hợp chia thành các nhóm $T$ có ít hơn hoặc bằng 3 người thích 1 giám khảo, khi đấy nếu trong những nhóm có nhiều hơn 11 người thích 1 giám khảo, ta sẽ "chia sẻ" sang nhóm $T$ (vì mỗi người thích ít nhất 10 giám khảo nên việc chia sẽ là hoàn toàn dễ dàng).
Bổ đề: phải có ít nhất 5 nhóm có ít nhất 4 người sao cho mỗi nhóm thích 1 giám khảo phân biệt.
Chứng minh: dễ dàng chứng minh bằng dirichlet
Trở lại bài toán: Vì theo bổ đề, ta có ít nhất 5 nhóm 4 người có thể thích 1 giám khảo phân biệt nên ta sẽ chia thành 5 nhóm 8 người và 20 nhóm 3 người. xét trong các nhóm 8 người, ta có thể chuyển nhiều nhất là 4 người khác sang các nhóm khác (vì 4 người này có thể không thích chung 1 giám khảo như 4 nguời còn lại), mà có thảy 5 nhóm cần chuyển người, nên ta chuyển đi nhiều nhất là 20 người và 20 người này hoàn toàn có thể chuyển đến nhiều nhất là 20 nhóm (mỗi người chuyển đến 1 nhóm). Nếu như tồn tại nhóm $T$ mà học sinh ở nhóm này thích cùng 1 vị giám khảo với 1 trong 5 nhóm "4 người" thì thì ta sẽ ghép nhóm này vào nhóm 4 người trên. Như vậy, ta đã xây dựng được nhiều nhất là 25 nhóm học sinh sao cho mỗi học sinh này thích 1 giám khảo khác nhau .
[RIGHT][I][B]Nguồn: MathScope.ORG[/B][/I][/RIGHT]
 
__________________
i'll try my best.

thay đổi nội dung bởi: quocbaoct10, 26-03-2015 lúc 10:24 PM
quocbaoct10 is offline   Trả Lời Với Trích Dẫn
The Following 3 Users Say Thank You to quocbaoct10 For This Useful Post:
dangvip123tb (01-04-2015), hoang_kkk (01-04-2015), huynhcongbang (27-03-2015)
Old 26-03-2015, 10:04 PM   #12
dinhngoctung
+Thành Viên+
 
Tham gia ngày: Mar 2015
Bài gởi: 1
Thanks: 0
Thanked 0 Times in 0 Posts
Bài hình đề gõ thiếu, 4 góc đều bằng alpha.
[RIGHT][I][B]Nguồn: MathScope.ORG[/B][/I][/RIGHT]
 
dinhngoctung is offline   Trả Lời Với Trích Dẫn
Old 26-03-2015, 10:50 PM   #13
Kelacloi
+Thành Viên+
 
Tham gia ngày: Mar 2011
Bài gởi: 252
Thanks: 50
Thanked 164 Times in 114 Posts
Hello mọi người, em xuất hiện trễ
Em bổ sung bài 1b hướng khác.
Đó là chứng minh 2 nhận xét:
Nhận xét 1:
"Nếu bộ $d_0,d_1,..,d_n \ge 0$ thoả:
i) $2015=d_0+d_1\alpha^1+d_2\alpha^2+..+d_n\alpha^n$
i) $d_0+..+d_n$ min
thì $ 0 \le d_k \le 4 \forall 0 \le k \le n$"

Nhận xét 2:
Nếu tồn tại 2 bộ số $(e_0,e_1,..,e_m)$ và $(d_0,d_1,..,d_n)$
thoả:
1) $ 0\le e_k \le 4 ; 0 \le d_k \le 4$
2) $d_0+d_1\alpha^1+d_2\alpha^2+..+d_n\alpha^n=e_0+e_ 1\alpha^1+e_2\alpha^2+..+e_m\alpha^m$
thì 2 bộ đó trùng nhau "
[RIGHT][I][B]Nguồn: MathScope.ORG[/B][/I][/RIGHT]
 
__________________
Kelacloi is offline   Trả Lời Với Trích Dẫn
The Following 3 Users Say Thank You to Kelacloi For This Useful Post:
dangvip123tb (01-04-2015), huynhcongbang (27-03-2015), vantienducdh (26-03-2015)
Old 26-03-2015, 11:53 PM   #14
Nguyen Van Linh
Moderator
 
Tham gia ngày: Aug 2009
Đến từ: Hà Nội
Bài gởi: 277
Thanks: 69
Thanked 323 Times in 145 Posts
Bài 5 vừa lạ vừa quen :3
a) Dễ thấy tứ giác $BEFC$ nội tiếp đồng thời $\angle APB=\angle APC=\angle AQE=\angle AQF=\alpha$ nên $AQ$ và $AP$ đẳng giác trong góc $BAC.$
Gọi $T'$ là điểm liên hợp đẳng giác của $Q$ trong tam giác $AEF$. Hiển nhiên $T'\in AT.$
Gọi $X$ là điểm đối xứng với $T'$ qua $EF$.
Ta có $\angle X'EF=\angle T'EF=\angle AEQ, \angle XFE=\angle T'FE=\angle AFQ$, suy ra $A$ và $X$ liên hợp đẳng giác trong tam giác $EQF$. Mà $QA$ là phân giác $\angle EQF$ nên $X,Q,A$ thẳng hàng.
Do đối xứng của $T'$ qua $EF$ nằm trên $QA$ nên $T'$ nằm trên đối xứng của $QA$ qua $EF$. Mà $D$ đối xứng với $Q$ qua $EF, DT$ là phân giác $\angle EDF$ nên $DT$ đối xứng với $QA$ qua $EF$. Từ đó $T'\equiv T.$
Suy ra $\angle DET=\angle XEQ=\angle FEA=\angle ABC$. Tương tự $\angle DFT=\angle ACB.$
b) Ta có $\angle PED=\angle DEF+\angle FEA+\angle PEA=\angle QEF+\angle ABC+\angle PBA=\angle ABC+\angle PBA+\angle PBC=2\angle BAC$. Theo câu a, $\angle DET=\angle ABC$ nên $ET$ là phân giác $\angle DEA$. Tương tự $FT$ là phân giác $\angle DFA$, suy ra tứ giác $DEPF$ ngoại tiếp đường tròn tâm $T.$
Qua $D$ kẻ tiếp tuyến thứ hai tới $(I)$, cắt $PT$ tại $L.$
Tứ giác $DEPL, DEPF$ ngoại tiếp nên $DF-PF=DE-PE=DL-PL$, suy ra $DLPF$ ngoại tiếp, hay $DL$ là tiếp tuyến chung của $(I)$ và $(J)$. Từ đó $\angle IDJ=\frac{1}{2}\angle EDF.$
Mặt khác, gọi $Y$ là tâm nội tiếp tam giác $DMN$, ta có $\angle IYJ=180^\circ-\angle MIN=90^\circ-\frac{1}{2}\angle MDN=\frac{1}{2}\angle EDF.$
Suy ra $D,Y,I,J$ nội tiếp. Hiển nhiên $\angle YDH=90^\circ$ nên $YH$ là đường kính của $(DIJ)$. Suy ra $YH$ đi qua $K.$
[RIGHT][I][B]Nguồn: MathScope.ORG[/B][/I][/RIGHT]
 
Hình Kèm Theo
Kiểu File : png P5.png (34.0 KB, 79 lần tải)

thay đổi nội dung bởi: Nguyen Van Linh, 27-03-2015 lúc 01:05 AM
Nguyen Van Linh is offline   Trả Lời Với Trích Dẫn
The Following 5 Users Say Thank You to Nguyen Van Linh For This Useful Post:
dangvip123tb (01-04-2015), huynhcongbang (27-03-2015), pco (27-03-2015), thaygiaocht (27-03-2015), thiendieu96 (27-03-2015)
Old 28-03-2015, 07:04 AM   #15
ThangToan
+Thành Viên+
 
Tham gia ngày: Nov 2010
Đến từ: THPT chuyên Vĩnh Phúc
Bài gởi: 570
Thanks: 24
Thanked 537 Times in 263 Posts
Trích:
Nguyên văn bởi Kelacloi View Post
Hello mọi người, em xuất hiện trễ
Em bổ sung bài 1b hướng khác.
Đó là chứng minh 2 nhận xét:
Nhận xét 1:
"Nếu bộ $d_0,d_1,..,d_n \ge 0$ thoả:
i) $2015=d_0+d_1\alpha^1+d_2\alpha^2+..+d_n\alpha^n$
i) $d_0+..+d_n$ min
thì $ 0 \le d_k \le 4 \forall 0 \le k \le n$"

Nhận xét 2:
Nếu tồn tại 2 bộ số $(e_0,e_1,..,e_m)$ và $(d_0,d_1,..,d_n)$
thoả:
1) $ 0\le e_k \le 4 ; 0 \le d_k \le 4$
2) $d_0+d_1\alpha^1+d_2\alpha^2+..+d_n\alpha^n=e_0+e_ 1\alpha^1+e_2\alpha^2+..+e_m\alpha^m$
thì 2 bộ đó trùng nhau "
Ta có thể làm cụ thể theo hướng này như sau:
Nhận xét 3.
$\begin{array}{l}
2015 = 5.403 = 403\left( {{\alpha ^2} + \alpha } \right) = 3\left( {{\alpha ^2} + \alpha } \right) + 16.25 = 3\left( {{\alpha ^2} + \alpha } \right) + 16{\left( {{\alpha ^2} + \alpha } \right)^2}\\
= 3\left( {{\alpha ^2} + \alpha } \right) + {\left( {{\alpha ^2} + \alpha } \right)^3} + 3{\left( {{\alpha ^2} + \alpha } \right)^4}\\
= {\alpha ^{11}} + 3{\alpha ^{10}} + 4{\alpha ^7} + 3{\alpha ^6} + {\alpha ^5} + {\alpha ^4} + {\alpha ^3} + 3{\alpha ^2} + 3\alpha
\end{array}$
Nhận xét 1:
"Nếu bộ $d_0,d_1,..,d_n \ge 0$ thoả:
i) $2015=d_0+d_1\alpha^1+d_2\alpha^2+..+d_n\alpha^n$
i) $d_0+..+d_n$ min
thì $ 0 \le d_k \le 4 \forall 0 \le k \le n$"
Chứng minh.
nếu tồn tại ${d_i} \ge 5 \Rightarrow {d_i} = 5 + {t_i},\,\,{t_i} \ge 0$
Khi đó $\begin{array}{l}
2015 = {d_0} + {d_1}\alpha + ... + \left( {5 + {t_i}} \right){\alpha ^i} + ... + {d_n}{\alpha ^n}\\
= {d_0} + {d_1}\alpha + ... + {t_i}{\alpha ^i} + ... + {d_n}{\alpha ^n} + 5{\alpha ^i}\\
= {d_0} + {d_1}\alpha + ... + {t_i}{\alpha ^i} + ... + {d_n}{\alpha ^n} + {\alpha ^i}\left( {\alpha + {\alpha ^2}} \right)\\
= {d_0} + {d_1}\alpha + ... + {t_i}{\alpha ^i} + ... + {d_n}{\alpha ^n} + {\alpha ^{i + 2}} + {\alpha ^{i + 1}}
\end{array}$
Do đó tổng các hệ số mới là:
$\begin{array}{l}
{d_0} + {d_1} + ... + {t_i} + {d_{i + 1}} + ... + {d_n} + 1 + 1 = {d_0} + {d_1} + ... + {t_i} + 2 + {d_{i + 1}} + ... + {d_n}\\
< {d_0} + {d_1} + ... + {t_i} + 5 + {d_{i + 1}} + ... + {d_n} = {d_0} + {d_1} + ... + {d_i} + {d_{i + 1}} + ... + {d_n}
\end{array}$
Do đó nhận xét 1 được chứng minh.
Nhận xét 2. Nếu ${d_i} \in Z,\,\,\left| {{d_i}} \right| \le 4,\,\,{d_0} + {d_1}\alpha + ... + {d_n}{\alpha ^n} = 0$ thì ${d_0} = {d_1} = {d_2} = ... = {d_n} = 0$
Thật vậy,
Xét đa thức $f\left( x \right) = {d_0} + {d_1}x + ... + {d_n}{x^n}$, lấy $f(x)$ chia cho đa thức $ x^2+x-5$ ta được:$\begin{array}{l}
f\left( x \right) = \left( {{x^2} + x - 5} \right)g\left( x \right) + ux + v\\
u,\,v \in Z,\,\,g\left( x \right) = {e_0} + {e_1}x + ... + {e_{n - 2}}{x^{n - 2}},\,\,{e_i} \in Z
\end{array}$
Lấy $\begin{array}{l}
x = \alpha \Rightarrow f\left( \alpha \right) = \left( {{\alpha ^2} + \alpha - 5} \right)g\left( \alpha \right) + u\alpha + v \Rightarrow u\alpha + v = 0 \Rightarrow u = v = 0\\
\Rightarrow f\left( x \right) = \left( {{x^2} + x - 5} \right)g\left( x \right)
\end{array}$
Đồng nhất hệ số ta được:
$\begin{array}{l}
{d_0} = - 5{e_0}\\
{d_1} = {e_0} - 5{e_1}\\
{d_2} = {e_0} + {e_1} - 5{e_2}\\
...\\
{d_{n - 2}} = {e_{n - 4}} + {e_{n - 3}} - 5{e_{n - 2}}\\
{d_{n - 1}} = {e_{n - 2}} + {e_{n - 1}}\\
{d_n} = {e_{n - 2}}
\end{array}$
Do $\begin{array}{l}
\left| {{d_0}} \right| \le 4,\,\,{d_0} = - 5{e_0} \Rightarrow {e_0} = {d_0} = 0\\
{d_1} = {e_0} - 5{e_1} = - 5{e_1} \Rightarrow {e_1} = {d_1} = 0\\
{d_2} = {e_0} + {e_1} - 5{e_2} = - 5{e_2} \Rightarrow {e_2} = {d_2} = 0\\
....\\
{d_n} = 0
\end{array}$
Do đó Nhận xét 2 được chứng minh.
Trở lại bài toán.
Giả sử ${c_0} + {c_1} + ... + {c_n}$ là nhỏ nhất. Khi đó theo nhận xét 1 ta được $0 \le {c_i} \le 4,\,\,i = 0,1,2,...,n$
Theo nhận xét 3 và nhận xét 2 ta được:
$\begin{array}{l}
{c_{12}} = {c_{13}} = ... = {c_n} = 0,\,\,{c_{11}} = 1,\,{c_{10}} = 3,{c_9} = {c_8} = 0,\,{c_7} = 4,{c_6} = 3,{c_5} = {c_4} = {c_3} = 1,{c_2} = {c_1} = 3,{c_0} = 0\\
\Rightarrow {c_0} + {c_1} + ... + {c_n} = 20.
\end{array}$
Vậy giá trị nhỏ nhất là 20.
[RIGHT][I][B]Nguồn: MathScope.ORG[/B][/I][/RIGHT]
 

thay đổi nội dung bởi: ThangToan, 28-03-2015 lúc 07:11 AM
ThangToan is offline   Trả Lời Với Trích Dẫn
The Following 4 Users Say Thank You to ThangToan For This Useful Post:
dangvip123tb (01-04-2015), huynhcongbang (28-03-2015), thaygiaocht (28-03-2015), vantienducdh (28-03-2015)
Trả lời Gởi Ðề Tài Mới

Bookmarks


Quuyền Hạn Của Bạn
You may not post new threads
You may not post replies
You may not post attachments
You may not edit your posts

BB code is Mở
Smilies đang Mở
[IMG] đang Mở
HTML đang Tắt

Chuyển đến


Múi giờ GMT. Hiện tại là 02:29 AM.


Powered by: vBulletin Copyright ©2000-2024, Jelsoft Enterprises Ltd.
Inactive Reminders By mathscope.org
[page compression: 125.98 k/144.01 k (12.52%)]